Q7

User avatar
 
ManhattanPrepLSAT1
Thanks Received: 1909
Atticus Finch
Atticus Finch
 
Posts: 2851
Joined: October 07th, 2009
 
 
 

Q7

by ManhattanPrepLSAT1 Sun Sep 25, 2016 1:49 am

Image
 
LaContiB630
Thanks Received: 0
Vinny Gambini
Vinny Gambini
 
Posts: 3
Joined: November 09th, 2017
 
 
 

Re: Q7

by LaContiB630 Wed Jan 31, 2018 8:40 am

Can someone explain the setup of this particular question? My answer choice was C, but the actual answer is E. Thank you.
User avatar
 
ohthatpatrick
Thanks Received: 3807
Atticus Finch
Atticus Finch
 
Posts: 4661
Joined: April 01st, 2011
 
 
 

Re: Q7

by ohthatpatrick Thu Feb 01, 2018 1:05 am

Because the question gives you a NEW RULE, as opposed to a specific placement (like "If Irises is fourth"), you want to:

1. write out the NEW RULE
2. look at the other rules and see if they connect at all

and then maybe
3. start framing some possible scenarios


We would write IG, and then look at our rules and see that rule 1 says that G is right before K.

Okay, so linking IG with GK gives us IGK.

The 3rd rule tells us that I and L have to be next to each other, so linking (IL) with IGK gives us LIGK.

That's a pretty unwieldy 4 person chunk. We also know from rule 2 that H is before K (but not first), so combining H - K with LIGK, we get
H - LIGK

Since H can't be 1st, the earliest it could go would be 2nd.
Since H has four things after it, the latest it could go would be 3rd.

So I might frame two scenarios: one with H in 2nd, one with H in 3rd.

Other people would simply judge the answer choices against
H - LIGK
M: 1, 2, or 3
and
F: 1 or 7

We can try it this second way.

(A) Putting G 4th means that you have H L I G K as your first five people. That breaks two rules: H can't be 1st, and we didn't save a spot for M to be 1, 2, or 3.

(B) H going 4th is impossible. We already figured out it must be 2 or 3. If it went 4th, there wouldn't be room for LIGK in the final three spots.

(C) Putting I 3rd means again that you have H L I G K, so this would break the same two rules that (A) was breaking.

(D) L is 2nd means again that you have H L I G K, so it's also breaking two rules.

(E) M is 3rd looks doable.
F H M L I G K is acceptable

If you're tempted by (C), write out a complete scenario as I just did for (E) and confirm that it doesn't break any rules.